AlexY297
Thanks Received: 0
Jackie Chiles
Jackie Chiles
 
Posts: 32
Joined: September 26th, 2018
 
 
 

Question #940

by AlexY297 Sun Dec 22, 2019 2:15 am

Hello I'm not sure I understand this problem completely. Then below it says "If V and R are together, it must be true that ..."

Can someone explain this please?

Thank you,
Alex
 
Laura Damone
Thanks Received: 93
Atticus Finch
Atticus Finch
 
Posts: 468
Joined: February 17th, 2011
 
 
 

Re: Question #940

by Laura Damone Tue Jan 07, 2020 6:18 pm

Hi there! This diagram is a grouping board. The groups are "m" and "t" - let's call them Monday and Tuesday, and in each group there are three slots. This means 3 elements go on Monday and 3 go on Tuesday. The rules tell us that V and W go together, and that if V is on Monday, R is on Tuesday.

The question stem asks us, if V and R go on the same day, which of the following must be true?

Well, if V and R are on the same day, the "V Monday --> R Tuesday" rule can't get triggered because it would split them up. That means the trigger condition, V on Monday, can't be met. V, then, must be on Tuesday. Since V must be with W and R, that fills up Tuesday and forces everybody else to go Monday, providing the correct answer: Y is on Mon.

Hope the helps!
Laura Damone
LSAT Content & Curriculum Lead | Manhattan Prep